Show this inequality with fractional parts.

The name of the pictureThe name of the pictureThe name of the pictureClash Royale CLAN TAG#URR8PPP











up vote
3
down vote

favorite
1












Let $n geq 2$ be an integer and $x_1, x_2, cdots, x_n$ positive reals such that $x_1x_2 cdots x_n = 1$.
Show: $$x_1 + x_2 + cdots + x_n < frac2n-12$$
Note: $x$ denotes the fractional part of $x$.



Is $dfrac2n-12$ optimal?







share|cite|improve this question






















  • Would you provide the source? And of course, your own efforts, please. Thank you!
    – A. Pongrácz
    Aug 16 at 10:04














up vote
3
down vote

favorite
1












Let $n geq 2$ be an integer and $x_1, x_2, cdots, x_n$ positive reals such that $x_1x_2 cdots x_n = 1$.
Show: $$x_1 + x_2 + cdots + x_n < frac2n-12$$
Note: $x$ denotes the fractional part of $x$.



Is $dfrac2n-12$ optimal?







share|cite|improve this question






















  • Would you provide the source? And of course, your own efforts, please. Thank you!
    – A. Pongrácz
    Aug 16 at 10:04












up vote
3
down vote

favorite
1









up vote
3
down vote

favorite
1






1





Let $n geq 2$ be an integer and $x_1, x_2, cdots, x_n$ positive reals such that $x_1x_2 cdots x_n = 1$.
Show: $$x_1 + x_2 + cdots + x_n < frac2n-12$$
Note: $x$ denotes the fractional part of $x$.



Is $dfrac2n-12$ optimal?







share|cite|improve this question














Let $n geq 2$ be an integer and $x_1, x_2, cdots, x_n$ positive reals such that $x_1x_2 cdots x_n = 1$.
Show: $$x_1 + x_2 + cdots + x_n < frac2n-12$$
Note: $x$ denotes the fractional part of $x$.



Is $dfrac2n-12$ optimal?









share|cite|improve this question













share|cite|improve this question




share|cite|improve this question








edited Aug 16 at 9:46









A. Pongrácz

3,827625




3,827625










asked Aug 16 at 9:36









communnites

1,209431




1,209431











  • Would you provide the source? And of course, your own efforts, please. Thank you!
    – A. Pongrácz
    Aug 16 at 10:04
















  • Would you provide the source? And of course, your own efforts, please. Thank you!
    – A. Pongrácz
    Aug 16 at 10:04















Would you provide the source? And of course, your own efforts, please. Thank you!
– A. Pongrácz
Aug 16 at 10:04




Would you provide the source? And of course, your own efforts, please. Thank you!
– A. Pongrácz
Aug 16 at 10:04










3 Answers
3






active

oldest

votes

















up vote
1
down vote



accepted
+50










Assume that $ngeq2$, $x_1x_2cdots x_n=1$, and
$$x_1+x_2+ldots+x_ngeq n-1over2 .tag1$$
Put $tau_i:=1-x_i>0$. Then $(1)$ implies $sum_itau_ileq1over2$; in particular no $tau_i$ is $=1$. We therefore may write $x_i=m_i-tau_i$ with $m_i=lceil x_irceilgeq1$. As $prod_i x_i=1$ at least one $m_i$ is $geq2$. Now
$$1=prod_i x_i=prod_i m_icdotprod_ileft(1-tau_iover m_iright) .tag2$$
We shall need the following Lemma, which is easily proven using induction: If $ngeq2$, $x_i>0$ $(1leq ileq n)$, and $sum_i x_i<1$ then
$$prod_i=1^n(1-x_i)>1-sum_i=1^n x_i .$$Since $sum_itau_iover m_i<1over2$ we then obtain from $(2)$ the contradiction
$$1geq 2left(1-sum_itau_iover m_iright)>2cdot1over2=1 .$$






share|cite|improve this answer



























    up vote
    0
    down vote













    The bound is certainly optimal (if true): let $x_1= frac12+varepsilon, x_n= 2-varepsilon$, and all other $x_i= 1-varepsilon$. (Not with the same number $varepsilon$, just pick very small positive reals like this.)



    This example also suggests the proof strategy.
    We may assume that none of the numbers are integers, as then a small perturbation leads to a significant improvement.
    Partition the indices so that $x_1, x_2, ldots, x_k< 1$ and the rest is greater than $1$.
    This is possible with reindexing the elements if necessary. Note that because of the condition, $1leq kleq n-1$.



    Assume that in the latter product, there are more than one terms that are greater than $2$. Then we can improve the objective function again: replace one such $x_i$ by $x_i-1$, and multiply all the numbers $x_1, x_2, ldots, x_k$ by $sqrt[k]fracx_ix_i-1$.
    We have that the first $k$ numbers are still eactly the ones below $1$. By successive application of this step, we may assume that only one of the numbers that are greater than $1$ is greater than $2$.



    A similar simplification can be made if $kgeq 2$. In that case, we can first replace $x_2$ by $1+varepsilon$ and $x_1$ by $x_1x_2/(1+varepsilon)$, iproving on the objective function.



    So we may assume that $x_1<1$, $1<x_2, ldots, x_n-1<2$ and $x_n>2$.



    Further hints: try to push those middle elements close to $1$ while improving on the objective function.



    Once you are done with that, the problem has essentially one varaible, as $x_1x_napprox 1$, and you can get rid of the fractional parts easily in the objective function, to make it a simple case of finding the maximum of a univariate function.






    share|cite|improve this answer





























      up vote
      0
      down vote













      Proof for $n=2$.



      One and only one between $lfloorx_1rfloor$ and $lfloorx_2rfloor$ should be equal to $0$ since $x_1x_2=1$ and if both $x_1$ and $x_2$ are nul the proof follows immediately.



      Put $x_1=k+x_1$ where $kinmathbb N^+$ and $x_1gt0$ so $x_2=x_2=dfrac1k+x_1$.



      We have $x_1+x_2=x_1+dfrac1k+x_1ltdfrac2cdot2-12=1+dfrac12spacecolorredlarge?$



      If $kge2$ then $x_1+dfrac1k+x_1lex_1+dfrac 12$ so $colorredtext YES$.



      It remains to prove for $k=1$. In this case if $x_1+dfrac11+x_1=dfrac32$ the positive number $x_1$ is the positive root of $$2X^2-X-1=0Rightarrowx_1=1,colorred text absurde$$
      A fortiori for $x_1+dfrac11+x_1gtdfrac32$ which end the proof.



      Can someone from this get the general answer for $ngt2$?






      share|cite|improve this answer






















        Your Answer




        StackExchange.ifUsing("editor", function ()
        return StackExchange.using("mathjaxEditing", function ()
        StackExchange.MarkdownEditor.creationCallbacks.add(function (editor, postfix)
        StackExchange.mathjaxEditing.prepareWmdForMathJax(editor, postfix, [["$", "$"], ["\\(","\\)"]]);
        );
        );
        , "mathjax-editing");

        StackExchange.ready(function()
        var channelOptions =
        tags: "".split(" "),
        id: "69"
        ;
        initTagRenderer("".split(" "), "".split(" "), channelOptions);

        StackExchange.using("externalEditor", function()
        // Have to fire editor after snippets, if snippets enabled
        if (StackExchange.settings.snippets.snippetsEnabled)
        StackExchange.using("snippets", function()
        createEditor();
        );

        else
        createEditor();

        );

        function createEditor()
        StackExchange.prepareEditor(
        heartbeatType: 'answer',
        convertImagesToLinks: true,
        noModals: false,
        showLowRepImageUploadWarning: true,
        reputationToPostImages: 10,
        bindNavPrevention: true,
        postfix: "",
        noCode: true, onDemand: true,
        discardSelector: ".discard-answer"
        ,immediatelyShowMarkdownHelp:true
        );



        );








         

        draft saved


        draft discarded


















        StackExchange.ready(
        function ()
        StackExchange.openid.initPostLogin('.new-post-login', 'https%3a%2f%2fmath.stackexchange.com%2fquestions%2f2884592%2fshow-this-inequality-with-fractional-parts%23new-answer', 'question_page');

        );

        Post as a guest






























        3 Answers
        3






        active

        oldest

        votes








        3 Answers
        3






        active

        oldest

        votes









        active

        oldest

        votes






        active

        oldest

        votes








        up vote
        1
        down vote



        accepted
        +50










        Assume that $ngeq2$, $x_1x_2cdots x_n=1$, and
        $$x_1+x_2+ldots+x_ngeq n-1over2 .tag1$$
        Put $tau_i:=1-x_i>0$. Then $(1)$ implies $sum_itau_ileq1over2$; in particular no $tau_i$ is $=1$. We therefore may write $x_i=m_i-tau_i$ with $m_i=lceil x_irceilgeq1$. As $prod_i x_i=1$ at least one $m_i$ is $geq2$. Now
        $$1=prod_i x_i=prod_i m_icdotprod_ileft(1-tau_iover m_iright) .tag2$$
        We shall need the following Lemma, which is easily proven using induction: If $ngeq2$, $x_i>0$ $(1leq ileq n)$, and $sum_i x_i<1$ then
        $$prod_i=1^n(1-x_i)>1-sum_i=1^n x_i .$$Since $sum_itau_iover m_i<1over2$ we then obtain from $(2)$ the contradiction
        $$1geq 2left(1-sum_itau_iover m_iright)>2cdot1over2=1 .$$






        share|cite|improve this answer
























          up vote
          1
          down vote



          accepted
          +50










          Assume that $ngeq2$, $x_1x_2cdots x_n=1$, and
          $$x_1+x_2+ldots+x_ngeq n-1over2 .tag1$$
          Put $tau_i:=1-x_i>0$. Then $(1)$ implies $sum_itau_ileq1over2$; in particular no $tau_i$ is $=1$. We therefore may write $x_i=m_i-tau_i$ with $m_i=lceil x_irceilgeq1$. As $prod_i x_i=1$ at least one $m_i$ is $geq2$. Now
          $$1=prod_i x_i=prod_i m_icdotprod_ileft(1-tau_iover m_iright) .tag2$$
          We shall need the following Lemma, which is easily proven using induction: If $ngeq2$, $x_i>0$ $(1leq ileq n)$, and $sum_i x_i<1$ then
          $$prod_i=1^n(1-x_i)>1-sum_i=1^n x_i .$$Since $sum_itau_iover m_i<1over2$ we then obtain from $(2)$ the contradiction
          $$1geq 2left(1-sum_itau_iover m_iright)>2cdot1over2=1 .$$






          share|cite|improve this answer






















            up vote
            1
            down vote



            accepted
            +50







            up vote
            1
            down vote



            accepted
            +50




            +50




            Assume that $ngeq2$, $x_1x_2cdots x_n=1$, and
            $$x_1+x_2+ldots+x_ngeq n-1over2 .tag1$$
            Put $tau_i:=1-x_i>0$. Then $(1)$ implies $sum_itau_ileq1over2$; in particular no $tau_i$ is $=1$. We therefore may write $x_i=m_i-tau_i$ with $m_i=lceil x_irceilgeq1$. As $prod_i x_i=1$ at least one $m_i$ is $geq2$. Now
            $$1=prod_i x_i=prod_i m_icdotprod_ileft(1-tau_iover m_iright) .tag2$$
            We shall need the following Lemma, which is easily proven using induction: If $ngeq2$, $x_i>0$ $(1leq ileq n)$, and $sum_i x_i<1$ then
            $$prod_i=1^n(1-x_i)>1-sum_i=1^n x_i .$$Since $sum_itau_iover m_i<1over2$ we then obtain from $(2)$ the contradiction
            $$1geq 2left(1-sum_itau_iover m_iright)>2cdot1over2=1 .$$






            share|cite|improve this answer












            Assume that $ngeq2$, $x_1x_2cdots x_n=1$, and
            $$x_1+x_2+ldots+x_ngeq n-1over2 .tag1$$
            Put $tau_i:=1-x_i>0$. Then $(1)$ implies $sum_itau_ileq1over2$; in particular no $tau_i$ is $=1$. We therefore may write $x_i=m_i-tau_i$ with $m_i=lceil x_irceilgeq1$. As $prod_i x_i=1$ at least one $m_i$ is $geq2$. Now
            $$1=prod_i x_i=prod_i m_icdotprod_ileft(1-tau_iover m_iright) .tag2$$
            We shall need the following Lemma, which is easily proven using induction: If $ngeq2$, $x_i>0$ $(1leq ileq n)$, and $sum_i x_i<1$ then
            $$prod_i=1^n(1-x_i)>1-sum_i=1^n x_i .$$Since $sum_itau_iover m_i<1over2$ we then obtain from $(2)$ the contradiction
            $$1geq 2left(1-sum_itau_iover m_iright)>2cdot1over2=1 .$$







            share|cite|improve this answer












            share|cite|improve this answer



            share|cite|improve this answer










            answered Aug 18 at 12:05









            Christian Blatter

            165k7109309




            165k7109309




















                up vote
                0
                down vote













                The bound is certainly optimal (if true): let $x_1= frac12+varepsilon, x_n= 2-varepsilon$, and all other $x_i= 1-varepsilon$. (Not with the same number $varepsilon$, just pick very small positive reals like this.)



                This example also suggests the proof strategy.
                We may assume that none of the numbers are integers, as then a small perturbation leads to a significant improvement.
                Partition the indices so that $x_1, x_2, ldots, x_k< 1$ and the rest is greater than $1$.
                This is possible with reindexing the elements if necessary. Note that because of the condition, $1leq kleq n-1$.



                Assume that in the latter product, there are more than one terms that are greater than $2$. Then we can improve the objective function again: replace one such $x_i$ by $x_i-1$, and multiply all the numbers $x_1, x_2, ldots, x_k$ by $sqrt[k]fracx_ix_i-1$.
                We have that the first $k$ numbers are still eactly the ones below $1$. By successive application of this step, we may assume that only one of the numbers that are greater than $1$ is greater than $2$.



                A similar simplification can be made if $kgeq 2$. In that case, we can first replace $x_2$ by $1+varepsilon$ and $x_1$ by $x_1x_2/(1+varepsilon)$, iproving on the objective function.



                So we may assume that $x_1<1$, $1<x_2, ldots, x_n-1<2$ and $x_n>2$.



                Further hints: try to push those middle elements close to $1$ while improving on the objective function.



                Once you are done with that, the problem has essentially one varaible, as $x_1x_napprox 1$, and you can get rid of the fractional parts easily in the objective function, to make it a simple case of finding the maximum of a univariate function.






                share|cite|improve this answer


























                  up vote
                  0
                  down vote













                  The bound is certainly optimal (if true): let $x_1= frac12+varepsilon, x_n= 2-varepsilon$, and all other $x_i= 1-varepsilon$. (Not with the same number $varepsilon$, just pick very small positive reals like this.)



                  This example also suggests the proof strategy.
                  We may assume that none of the numbers are integers, as then a small perturbation leads to a significant improvement.
                  Partition the indices so that $x_1, x_2, ldots, x_k< 1$ and the rest is greater than $1$.
                  This is possible with reindexing the elements if necessary. Note that because of the condition, $1leq kleq n-1$.



                  Assume that in the latter product, there are more than one terms that are greater than $2$. Then we can improve the objective function again: replace one such $x_i$ by $x_i-1$, and multiply all the numbers $x_1, x_2, ldots, x_k$ by $sqrt[k]fracx_ix_i-1$.
                  We have that the first $k$ numbers are still eactly the ones below $1$. By successive application of this step, we may assume that only one of the numbers that are greater than $1$ is greater than $2$.



                  A similar simplification can be made if $kgeq 2$. In that case, we can first replace $x_2$ by $1+varepsilon$ and $x_1$ by $x_1x_2/(1+varepsilon)$, iproving on the objective function.



                  So we may assume that $x_1<1$, $1<x_2, ldots, x_n-1<2$ and $x_n>2$.



                  Further hints: try to push those middle elements close to $1$ while improving on the objective function.



                  Once you are done with that, the problem has essentially one varaible, as $x_1x_napprox 1$, and you can get rid of the fractional parts easily in the objective function, to make it a simple case of finding the maximum of a univariate function.






                  share|cite|improve this answer
























                    up vote
                    0
                    down vote










                    up vote
                    0
                    down vote









                    The bound is certainly optimal (if true): let $x_1= frac12+varepsilon, x_n= 2-varepsilon$, and all other $x_i= 1-varepsilon$. (Not with the same number $varepsilon$, just pick very small positive reals like this.)



                    This example also suggests the proof strategy.
                    We may assume that none of the numbers are integers, as then a small perturbation leads to a significant improvement.
                    Partition the indices so that $x_1, x_2, ldots, x_k< 1$ and the rest is greater than $1$.
                    This is possible with reindexing the elements if necessary. Note that because of the condition, $1leq kleq n-1$.



                    Assume that in the latter product, there are more than one terms that are greater than $2$. Then we can improve the objective function again: replace one such $x_i$ by $x_i-1$, and multiply all the numbers $x_1, x_2, ldots, x_k$ by $sqrt[k]fracx_ix_i-1$.
                    We have that the first $k$ numbers are still eactly the ones below $1$. By successive application of this step, we may assume that only one of the numbers that are greater than $1$ is greater than $2$.



                    A similar simplification can be made if $kgeq 2$. In that case, we can first replace $x_2$ by $1+varepsilon$ and $x_1$ by $x_1x_2/(1+varepsilon)$, iproving on the objective function.



                    So we may assume that $x_1<1$, $1<x_2, ldots, x_n-1<2$ and $x_n>2$.



                    Further hints: try to push those middle elements close to $1$ while improving on the objective function.



                    Once you are done with that, the problem has essentially one varaible, as $x_1x_napprox 1$, and you can get rid of the fractional parts easily in the objective function, to make it a simple case of finding the maximum of a univariate function.






                    share|cite|improve this answer














                    The bound is certainly optimal (if true): let $x_1= frac12+varepsilon, x_n= 2-varepsilon$, and all other $x_i= 1-varepsilon$. (Not with the same number $varepsilon$, just pick very small positive reals like this.)



                    This example also suggests the proof strategy.
                    We may assume that none of the numbers are integers, as then a small perturbation leads to a significant improvement.
                    Partition the indices so that $x_1, x_2, ldots, x_k< 1$ and the rest is greater than $1$.
                    This is possible with reindexing the elements if necessary. Note that because of the condition, $1leq kleq n-1$.



                    Assume that in the latter product, there are more than one terms that are greater than $2$. Then we can improve the objective function again: replace one such $x_i$ by $x_i-1$, and multiply all the numbers $x_1, x_2, ldots, x_k$ by $sqrt[k]fracx_ix_i-1$.
                    We have that the first $k$ numbers are still eactly the ones below $1$. By successive application of this step, we may assume that only one of the numbers that are greater than $1$ is greater than $2$.



                    A similar simplification can be made if $kgeq 2$. In that case, we can first replace $x_2$ by $1+varepsilon$ and $x_1$ by $x_1x_2/(1+varepsilon)$, iproving on the objective function.



                    So we may assume that $x_1<1$, $1<x_2, ldots, x_n-1<2$ and $x_n>2$.



                    Further hints: try to push those middle elements close to $1$ while improving on the objective function.



                    Once you are done with that, the problem has essentially one varaible, as $x_1x_napprox 1$, and you can get rid of the fractional parts easily in the objective function, to make it a simple case of finding the maximum of a univariate function.







                    share|cite|improve this answer














                    share|cite|improve this answer



                    share|cite|improve this answer








                    edited Aug 16 at 10:33

























                    answered Aug 16 at 10:17









                    A. Pongrácz

                    3,827625




                    3,827625




















                        up vote
                        0
                        down vote













                        Proof for $n=2$.



                        One and only one between $lfloorx_1rfloor$ and $lfloorx_2rfloor$ should be equal to $0$ since $x_1x_2=1$ and if both $x_1$ and $x_2$ are nul the proof follows immediately.



                        Put $x_1=k+x_1$ where $kinmathbb N^+$ and $x_1gt0$ so $x_2=x_2=dfrac1k+x_1$.



                        We have $x_1+x_2=x_1+dfrac1k+x_1ltdfrac2cdot2-12=1+dfrac12spacecolorredlarge?$



                        If $kge2$ then $x_1+dfrac1k+x_1lex_1+dfrac 12$ so $colorredtext YES$.



                        It remains to prove for $k=1$. In this case if $x_1+dfrac11+x_1=dfrac32$ the positive number $x_1$ is the positive root of $$2X^2-X-1=0Rightarrowx_1=1,colorred text absurde$$
                        A fortiori for $x_1+dfrac11+x_1gtdfrac32$ which end the proof.



                        Can someone from this get the general answer for $ngt2$?






                        share|cite|improve this answer


























                          up vote
                          0
                          down vote













                          Proof for $n=2$.



                          One and only one between $lfloorx_1rfloor$ and $lfloorx_2rfloor$ should be equal to $0$ since $x_1x_2=1$ and if both $x_1$ and $x_2$ are nul the proof follows immediately.



                          Put $x_1=k+x_1$ where $kinmathbb N^+$ and $x_1gt0$ so $x_2=x_2=dfrac1k+x_1$.



                          We have $x_1+x_2=x_1+dfrac1k+x_1ltdfrac2cdot2-12=1+dfrac12spacecolorredlarge?$



                          If $kge2$ then $x_1+dfrac1k+x_1lex_1+dfrac 12$ so $colorredtext YES$.



                          It remains to prove for $k=1$. In this case if $x_1+dfrac11+x_1=dfrac32$ the positive number $x_1$ is the positive root of $$2X^2-X-1=0Rightarrowx_1=1,colorred text absurde$$
                          A fortiori for $x_1+dfrac11+x_1gtdfrac32$ which end the proof.



                          Can someone from this get the general answer for $ngt2$?






                          share|cite|improve this answer
























                            up vote
                            0
                            down vote










                            up vote
                            0
                            down vote









                            Proof for $n=2$.



                            One and only one between $lfloorx_1rfloor$ and $lfloorx_2rfloor$ should be equal to $0$ since $x_1x_2=1$ and if both $x_1$ and $x_2$ are nul the proof follows immediately.



                            Put $x_1=k+x_1$ where $kinmathbb N^+$ and $x_1gt0$ so $x_2=x_2=dfrac1k+x_1$.



                            We have $x_1+x_2=x_1+dfrac1k+x_1ltdfrac2cdot2-12=1+dfrac12spacecolorredlarge?$



                            If $kge2$ then $x_1+dfrac1k+x_1lex_1+dfrac 12$ so $colorredtext YES$.



                            It remains to prove for $k=1$. In this case if $x_1+dfrac11+x_1=dfrac32$ the positive number $x_1$ is the positive root of $$2X^2-X-1=0Rightarrowx_1=1,colorred text absurde$$
                            A fortiori for $x_1+dfrac11+x_1gtdfrac32$ which end the proof.



                            Can someone from this get the general answer for $ngt2$?






                            share|cite|improve this answer














                            Proof for $n=2$.



                            One and only one between $lfloorx_1rfloor$ and $lfloorx_2rfloor$ should be equal to $0$ since $x_1x_2=1$ and if both $x_1$ and $x_2$ are nul the proof follows immediately.



                            Put $x_1=k+x_1$ where $kinmathbb N^+$ and $x_1gt0$ so $x_2=x_2=dfrac1k+x_1$.



                            We have $x_1+x_2=x_1+dfrac1k+x_1ltdfrac2cdot2-12=1+dfrac12spacecolorredlarge?$



                            If $kge2$ then $x_1+dfrac1k+x_1lex_1+dfrac 12$ so $colorredtext YES$.



                            It remains to prove for $k=1$. In this case if $x_1+dfrac11+x_1=dfrac32$ the positive number $x_1$ is the positive root of $$2X^2-X-1=0Rightarrowx_1=1,colorred text absurde$$
                            A fortiori for $x_1+dfrac11+x_1gtdfrac32$ which end the proof.



                            Can someone from this get the general answer for $ngt2$?







                            share|cite|improve this answer














                            share|cite|improve this answer



                            share|cite|improve this answer








                            edited Aug 16 at 18:52

























                            answered Aug 16 at 16:59









                            Piquito

                            17.4k31334




                            17.4k31334






















                                 

                                draft saved


                                draft discarded


























                                 


                                draft saved


                                draft discarded














                                StackExchange.ready(
                                function ()
                                StackExchange.openid.initPostLogin('.new-post-login', 'https%3a%2f%2fmath.stackexchange.com%2fquestions%2f2884592%2fshow-this-inequality-with-fractional-parts%23new-answer', 'question_page');

                                );

                                Post as a guest













































































                                這個網誌中的熱門文章

                                Is there any way to eliminate the singular point to solve this integral by hand or by approximations?

                                Why am i infinitely getting the same tweet with the Twitter Search API?

                                Carbon dioxide